Largest value of determinantProving the determinant on the L.H.S = determinant on the R.H.S.Prove that the...

Question about かな and だろう

How come there are so many candidates for the 2020 Democratic party presidential nomination?

Interpret a multiple linear regression when Y is log transformed

Minor Revision with suggestion of an alternative proof by reviewer

Could moose/elk survive in the Amazon forest?

Why isn't the definition of absolute value applied when squaring a radical containing a variable?

French for 'It must be my imagination'?

Can SQL Server create collisions in system generated constraint names?

How can I practically buy stocks?

What's the polite way to say "I need to urinate"?

Mjolnir's timeline from Thor's perspective

What makes accurate emulation of old systems a difficult task?

In order to check if a field is required or not, is the result of isNillable method sufficient?

What does a straight horizontal line above a few notes, after a changed tempo mean?

How to reduce LED flash rate (frequency)

Is it idiomatic to construct against `this`?

What are the potential pitfalls when using metals as a currency?

Phrase for the opposite of "foolproof"

Fizzy, soft, pop and still drinks

Why was primality test thought to be NP?

Rivers without rain

Does holding a wand and speaking its command word count as V/S/M spell components?

Refer to page numbers where table is referenced

Map of water taps to fill bottles



Largest value of determinant


Proving the determinant on the L.H.S = determinant on the R.H.S.Prove that the determinant is $0$ by expressing as a productLeast value of a determinantWrite the given Determinant as the product of two Determinantsdeterminant diagonal zero symmetric matrixSolving $n$th order determinantMatrix Determinant CalculationProve determinant of a matrix with trigonometry functions.Proving that two determinants are equal without expanding themShow the following matrix has determinant = 0













3












$begingroup$



If $alpha,beta,gamma in [-3,10].$ Then largest value of the determinant



$$begin{vmatrix}3alpha^2&beta^2+alphabeta+alpha^2&gamma^2+alphagamma+alpha^2\\
alpha^2+alphabeta+beta^2& 3beta^2&gamma^2+betagamma+beta^2\\
alpha^2+alphagamma+gamma^2& beta^2+betagamma+gamma^2&3gamma^2end{vmatrix}$$




Try: I am trying to break that determinant into product of 2 determinants but not able to break it.



Could someone help me in this question? Thanks.










share|cite|improve this question











$endgroup$

















    3












    $begingroup$



    If $alpha,beta,gamma in [-3,10].$ Then largest value of the determinant



    $$begin{vmatrix}3alpha^2&beta^2+alphabeta+alpha^2&gamma^2+alphagamma+alpha^2\\
    alpha^2+alphabeta+beta^2& 3beta^2&gamma^2+betagamma+beta^2\\
    alpha^2+alphagamma+gamma^2& beta^2+betagamma+gamma^2&3gamma^2end{vmatrix}$$




    Try: I am trying to break that determinant into product of 2 determinants but not able to break it.



    Could someone help me in this question? Thanks.










    share|cite|improve this question











    $endgroup$















      3












      3








      3





      $begingroup$



      If $alpha,beta,gamma in [-3,10].$ Then largest value of the determinant



      $$begin{vmatrix}3alpha^2&beta^2+alphabeta+alpha^2&gamma^2+alphagamma+alpha^2\\
      alpha^2+alphabeta+beta^2& 3beta^2&gamma^2+betagamma+beta^2\\
      alpha^2+alphagamma+gamma^2& beta^2+betagamma+gamma^2&3gamma^2end{vmatrix}$$




      Try: I am trying to break that determinant into product of 2 determinants but not able to break it.



      Could someone help me in this question? Thanks.










      share|cite|improve this question











      $endgroup$





      If $alpha,beta,gamma in [-3,10].$ Then largest value of the determinant



      $$begin{vmatrix}3alpha^2&beta^2+alphabeta+alpha^2&gamma^2+alphagamma+alpha^2\\
      alpha^2+alphabeta+beta^2& 3beta^2&gamma^2+betagamma+beta^2\\
      alpha^2+alphagamma+gamma^2& beta^2+betagamma+gamma^2&3gamma^2end{vmatrix}$$




      Try: I am trying to break that determinant into product of 2 determinants but not able to break it.



      Could someone help me in this question? Thanks.







      linear-algebra determinant






      share|cite|improve this question















      share|cite|improve this question













      share|cite|improve this question




      share|cite|improve this question








      edited 1 hour ago









      StubbornAtom

      6,74931442




      6,74931442










      asked 1 hour ago









      DXTDXT

      5,9812733




      5,9812733






















          2 Answers
          2






          active

          oldest

          votes


















          3












          $begingroup$

          Hint: Using the rules of SARRUS we get after simplifying $$- left( alpha-gamma right) ^{2} left( beta-gamma right) ^{2}
          left( beta-alpha right) ^{2}
          $$






          share|cite|improve this answer









          $endgroup$













          • $begingroup$
            I think so! What result do you get?
            $endgroup$
            – Dr. Sonnhard Graubner
            58 mins ago










          • $begingroup$
            This is nice to hear!
            $endgroup$
            – Dr. Sonnhard Graubner
            56 mins ago










          • $begingroup$
            A big typo is only a typo, big or not!
            $endgroup$
            – Dr. Sonnhard Graubner
            55 mins ago



















          1












          $begingroup$

          Hint: The matrix is symmetric. You can use the Cholesky decomposition to rewrite the matrix $A$ as $A=LL^T$, in which $L$ is a lower triangular matrix. Then the determinant is $det A = det L det L^T = det^2 L$. Then $det L$ is given by the product of the diagonal elements of $L$ as it is a lower triangular matrix.






          share|cite|improve this answer









          $endgroup$














            Your Answer








            StackExchange.ready(function() {
            var channelOptions = {
            tags: "".split(" "),
            id: "69"
            };
            initTagRenderer("".split(" "), "".split(" "), channelOptions);

            StackExchange.using("externalEditor", function() {
            // Have to fire editor after snippets, if snippets enabled
            if (StackExchange.settings.snippets.snippetsEnabled) {
            StackExchange.using("snippets", function() {
            createEditor();
            });
            }
            else {
            createEditor();
            }
            });

            function createEditor() {
            StackExchange.prepareEditor({
            heartbeatType: 'answer',
            autoActivateHeartbeat: false,
            convertImagesToLinks: true,
            noModals: true,
            showLowRepImageUploadWarning: true,
            reputationToPostImages: 10,
            bindNavPrevention: true,
            postfix: "",
            imageUploader: {
            brandingHtml: "Powered by u003ca class="icon-imgur-white" href="https://imgur.com/"u003eu003c/au003e",
            contentPolicyHtml: "User contributions licensed under u003ca href="https://creativecommons.org/licenses/by-sa/3.0/"u003ecc by-sa 3.0 with attribution requiredu003c/au003e u003ca href="https://stackoverflow.com/legal/content-policy"u003e(content policy)u003c/au003e",
            allowUrls: true
            },
            noCode: true, onDemand: true,
            discardSelector: ".discard-answer"
            ,immediatelyShowMarkdownHelp:true
            });


            }
            });














            draft saved

            draft discarded


















            StackExchange.ready(
            function () {
            StackExchange.openid.initPostLogin('.new-post-login', 'https%3a%2f%2fmath.stackexchange.com%2fquestions%2f3204547%2flargest-value-of-determinant%23new-answer', 'question_page');
            }
            );

            Post as a guest















            Required, but never shown

























            2 Answers
            2






            active

            oldest

            votes








            2 Answers
            2






            active

            oldest

            votes









            active

            oldest

            votes






            active

            oldest

            votes









            3












            $begingroup$

            Hint: Using the rules of SARRUS we get after simplifying $$- left( alpha-gamma right) ^{2} left( beta-gamma right) ^{2}
            left( beta-alpha right) ^{2}
            $$






            share|cite|improve this answer









            $endgroup$













            • $begingroup$
              I think so! What result do you get?
              $endgroup$
              – Dr. Sonnhard Graubner
              58 mins ago










            • $begingroup$
              This is nice to hear!
              $endgroup$
              – Dr. Sonnhard Graubner
              56 mins ago










            • $begingroup$
              A big typo is only a typo, big or not!
              $endgroup$
              – Dr. Sonnhard Graubner
              55 mins ago
















            3












            $begingroup$

            Hint: Using the rules of SARRUS we get after simplifying $$- left( alpha-gamma right) ^{2} left( beta-gamma right) ^{2}
            left( beta-alpha right) ^{2}
            $$






            share|cite|improve this answer









            $endgroup$













            • $begingroup$
              I think so! What result do you get?
              $endgroup$
              – Dr. Sonnhard Graubner
              58 mins ago










            • $begingroup$
              This is nice to hear!
              $endgroup$
              – Dr. Sonnhard Graubner
              56 mins ago










            • $begingroup$
              A big typo is only a typo, big or not!
              $endgroup$
              – Dr. Sonnhard Graubner
              55 mins ago














            3












            3








            3





            $begingroup$

            Hint: Using the rules of SARRUS we get after simplifying $$- left( alpha-gamma right) ^{2} left( beta-gamma right) ^{2}
            left( beta-alpha right) ^{2}
            $$






            share|cite|improve this answer









            $endgroup$



            Hint: Using the rules of SARRUS we get after simplifying $$- left( alpha-gamma right) ^{2} left( beta-gamma right) ^{2}
            left( beta-alpha right) ^{2}
            $$







            share|cite|improve this answer












            share|cite|improve this answer



            share|cite|improve this answer










            answered 1 hour ago









            Dr. Sonnhard GraubnerDr. Sonnhard Graubner

            79.9k42867




            79.9k42867












            • $begingroup$
              I think so! What result do you get?
              $endgroup$
              – Dr. Sonnhard Graubner
              58 mins ago










            • $begingroup$
              This is nice to hear!
              $endgroup$
              – Dr. Sonnhard Graubner
              56 mins ago










            • $begingroup$
              A big typo is only a typo, big or not!
              $endgroup$
              – Dr. Sonnhard Graubner
              55 mins ago


















            • $begingroup$
              I think so! What result do you get?
              $endgroup$
              – Dr. Sonnhard Graubner
              58 mins ago










            • $begingroup$
              This is nice to hear!
              $endgroup$
              – Dr. Sonnhard Graubner
              56 mins ago










            • $begingroup$
              A big typo is only a typo, big or not!
              $endgroup$
              – Dr. Sonnhard Graubner
              55 mins ago
















            $begingroup$
            I think so! What result do you get?
            $endgroup$
            – Dr. Sonnhard Graubner
            58 mins ago




            $begingroup$
            I think so! What result do you get?
            $endgroup$
            – Dr. Sonnhard Graubner
            58 mins ago












            $begingroup$
            This is nice to hear!
            $endgroup$
            – Dr. Sonnhard Graubner
            56 mins ago




            $begingroup$
            This is nice to hear!
            $endgroup$
            – Dr. Sonnhard Graubner
            56 mins ago












            $begingroup$
            A big typo is only a typo, big or not!
            $endgroup$
            – Dr. Sonnhard Graubner
            55 mins ago




            $begingroup$
            A big typo is only a typo, big or not!
            $endgroup$
            – Dr. Sonnhard Graubner
            55 mins ago











            1












            $begingroup$

            Hint: The matrix is symmetric. You can use the Cholesky decomposition to rewrite the matrix $A$ as $A=LL^T$, in which $L$ is a lower triangular matrix. Then the determinant is $det A = det L det L^T = det^2 L$. Then $det L$ is given by the product of the diagonal elements of $L$ as it is a lower triangular matrix.






            share|cite|improve this answer









            $endgroup$


















              1












              $begingroup$

              Hint: The matrix is symmetric. You can use the Cholesky decomposition to rewrite the matrix $A$ as $A=LL^T$, in which $L$ is a lower triangular matrix. Then the determinant is $det A = det L det L^T = det^2 L$. Then $det L$ is given by the product of the diagonal elements of $L$ as it is a lower triangular matrix.






              share|cite|improve this answer









              $endgroup$
















                1












                1








                1





                $begingroup$

                Hint: The matrix is symmetric. You can use the Cholesky decomposition to rewrite the matrix $A$ as $A=LL^T$, in which $L$ is a lower triangular matrix. Then the determinant is $det A = det L det L^T = det^2 L$. Then $det L$ is given by the product of the diagonal elements of $L$ as it is a lower triangular matrix.






                share|cite|improve this answer









                $endgroup$



                Hint: The matrix is symmetric. You can use the Cholesky decomposition to rewrite the matrix $A$ as $A=LL^T$, in which $L$ is a lower triangular matrix. Then the determinant is $det A = det L det L^T = det^2 L$. Then $det L$ is given by the product of the diagonal elements of $L$ as it is a lower triangular matrix.







                share|cite|improve this answer












                share|cite|improve this answer



                share|cite|improve this answer










                answered 1 hour ago









                MachineLearnerMachineLearner

                1,906213




                1,906213






























                    draft saved

                    draft discarded




















































                    Thanks for contributing an answer to Mathematics Stack Exchange!


                    • Please be sure to answer the question. Provide details and share your research!

                    But avoid



                    • Asking for help, clarification, or responding to other answers.

                    • Making statements based on opinion; back them up with references or personal experience.


                    Use MathJax to format equations. MathJax reference.


                    To learn more, see our tips on writing great answers.




                    draft saved


                    draft discarded














                    StackExchange.ready(
                    function () {
                    StackExchange.openid.initPostLogin('.new-post-login', 'https%3a%2f%2fmath.stackexchange.com%2fquestions%2f3204547%2flargest-value-of-determinant%23new-answer', 'question_page');
                    }
                    );

                    Post as a guest















                    Required, but never shown





















































                    Required, but never shown














                    Required, but never shown












                    Required, but never shown







                    Required, but never shown

































                    Required, but never shown














                    Required, but never shown












                    Required, but never shown







                    Required, but never shown







                    Popular posts from this blog

                    Щит и меч (фильм) Содержание Названия серий | Сюжет |...

                    is 'sed' thread safeWhat should someone know about using Python scripts in the shell?Nexenta bash script uses...

                    Meter-Bus Содержание Параметры шины | Стандартизация |...